closed by
1,207 views
2 votes
2 votes
closed as a duplicate of: maths_mocktest1_30
Let G be a finite group.If A and B are subgroups of G with orders 4 and 5 respectively then |A$\cap$B|=......
closed by

Related questions

0 votes
0 votes
0 answers
1
Nandkishor3939 asked Jan 13, 2019
892 views
Is this monoid:Addition modulo (take mode using m) on the set of Integers (Z m)={0,1,2,3,4,…..m-1}i.e. For all a a (+ modulo using m) e = e (+ modulo using m) a...
0 votes
0 votes
0 answers
2
jatin khachane 1 asked Dec 26, 2018
531 views
Is Every Group of Order $P^{k}$ such that P is prime and K is positive integer ABELIAN
6 votes
6 votes
2 answers
3
Na462 asked Dec 8, 2018
3,141 views
Let (G,*) be a group such that O(G) = 8, where O(G) denotes the order of the group. Which of the following is True ?There exist no element a in G whose order is 6.There e...
0 votes
0 votes
1 answer
4